Multivariable Triple Integral - Calculus Physics/Math Problem

Click For Summary
The discussion focuses on evaluating the moment of inertia of a homogeneous solid bounded by the equation (x^2+y^2+z^2)^2<=z. The original poster has expanded the area but is uncertain about the next steps in the integration process. Participants suggest clarifying whether to use cylindrical or spherical coordinates and emphasize the importance of verifying the Jacobian value. Additionally, they request to see the calculations performed so far to provide more effective assistance. The conversation highlights the need for clear communication of methods in solving complex calculus problems.
aligator11
Messages
10
Reaction score
0
Homework Statement
Evaluate the Moment of Inertia with respect to Oz axis of the [...] solid A: (x^2+y^2+z^2)^2<=z
Relevant Equations
Formula for moment of inertia: Iz=SSS(x^2+y^2)*constant-density*dxdydz

Cylinder Jacobian: SSS(x,y,z)*|r|dθdrdφ

Cylinder Jacobian substitution parameters:
x=rsin(θ)cos(φ)
y=rsin(θ)sin(φ)
z=rcos(θ)
Jacobian -->|r|=r^2*cos(θ)
Hello everybody.

If anyone could help me solve the calculus problem posted below, I would be greatful.

Task: Evaluate the moment of inertia with respect to Oz axis of the homogeneous solid A

Bounded by area - A: (x^2+y^2+z^2)^2<=zSo far I was able to expand A: [...] so that I receive something like this: r=(cos(θ))^(1/3) for the definite intergral of my radius. I'm not sure what is the next step I should take...

Thank you all the great souls which are able to help me in that matter.

Cheers!
 
Physics news on Phys.org
Well, first of all, do you want to integrate using cylindrical coordinates? Or using spherical coordinates? Because you say cylindrical but the variables you define are spherical, so think about this. Also, check the value you get for the Jacobian.

For the other part, I cannot help you if you don't show me what you did to get your answer, sorry.
 
Question: A clock's minute hand has length 4 and its hour hand has length 3. What is the distance between the tips at the moment when it is increasing most rapidly?(Putnam Exam Question) Answer: Making assumption that both the hands moves at constant angular velocities, the answer is ## \sqrt{7} .## But don't you think this assumption is somewhat doubtful and wrong?

Similar threads

  • · Replies 4 ·
Replies
4
Views
2K
  • · Replies 9 ·
Replies
9
Views
2K
  • · Replies 1 ·
Replies
1
Views
2K
  • · Replies 5 ·
Replies
5
Views
2K
  • · Replies 3 ·
Replies
3
Views
4K
  • · Replies 2 ·
Replies
2
Views
2K
  • · Replies 6 ·
Replies
6
Views
2K
Replies
6
Views
2K
  • · Replies 10 ·
Replies
10
Views
3K
  • · Replies 15 ·
Replies
15
Views
2K